Latest Banking jobs   »   NIACL AO Mains Exam Reasoning Quiz:...

NIACL AO Mains Exam Reasoning Quiz: 23rd February 2019

Dear Aspirants,

https://store.adda247.com/product-books/1723/Bank-Publication-Prime-for-PO-and-Clerk-In-English-Printed-Edition?utm_source=bankersadda&utm_medium=blog&utm_campaign=Bank%20Publications%20Prime



Reasoning Questions for NIACL AO PHASE II Exam 2019:

Reasoning Ability is an onerous section. With the increasing complexity of questions, it becomes hard for one to give it the cold shoulder. The only way to make the grade in this particular section in the forthcoming banking exams like NIACL AO PHASE II is to practice continuously with all your heart and soul. And, to let you practice with the best of the latest pattern questions, here is the Adda247 Reasoning Quiz based on the exact same pattern of questions that are being asked in the exams.



Directions (1–5): Study the following information to answer the given questions:
Twelve people are sitting in two parallel rows containing six people each, in such a way that there is an equal distance between adjacent persons. In row 1, S, N, Y, A, U and D are seated and all of them are facing South, and in row 2 H, T, F, L, I and G are sitting and all of them are facing North. Therefore, in the given seating arrangement each member seated in a row faces another member of the other row. H sits third to right of L. Neither H nor L sits at extreme ends. U faces L. I does not face D. Two persons sit between Y and N. Y is not an immediate neighbor of U. F does not face D. S is not an immediate neighbour of Y.D does not face H and D does not sit at any of the extreme ends. D is not an immediate neighbor of U. T sits at one of the extreme ends. Only two people sit between T and I.
Q1. Who amongst the following sit at extreme ends of the rows?
T, I
A, U
S, Y
T, G
None of these
Solution:
NIACL AO Mains Exam Reasoning Quiz: 23rd February 2019 |_3.1
Q2. Who amongst the following faces H?
Y
U
S
N
A
Solution:
NIACL AO Mains Exam Reasoning Quiz: 23rd February 2019 |_3.1
Q3. How many persons are seated between U and A?
One
Two
Three
Four
None
Solution:
NIACL AO Mains Exam Reasoning Quiz: 23rd February 2019 |_3.1
Q4. S is related to D in the same way as F is related to G. Which of the following is I related to, following the same pattern?
T
L
F
H
None of these
Solution:
NIACL AO Mains Exam Reasoning Quiz: 23rd February 2019 |_3.1
Q5. Which of the following is true regarding G?
G sits second to right of F.
G is not an immediate neighbour of H.
G sits third to left of L
G sits at one of the extreme ends of the line.
G faces D.
Solution:
NIACL AO Mains Exam Reasoning Quiz: 23rd February 2019 |_3.1
Directions (6-10): Study the following information carefully and answer the questions which follow–
In a certain code language
‘chitrakoot train derailed district’ is coded as ’56 83 71 37’,
‘Pradesh platform train accident’ is coded as ’76 75 38 83’,
‘early derailed platform hours’ is coded as ’37 86 16 76’ and
‘early accident of uttar’ is coded as ’75 58 16 63’.
Q6. What does ‘86’ stand for?
early
hours
derailed
platform
Cannot be determined
Solution:
NIACL AO Mains Exam Reasoning Quiz: 23rd February 2019 |_8.1
Q7. What can be the code for ‘new accident’?
38 76
75 56
71 38
51 75
Cannot be determined
Solution:
NIACL AO Mains Exam Reasoning Quiz: 23rd February 2019 |_8.1
Q8. Which of the following could be a code for ‘it platform early accident’?
75 36 86 76
75 16 38 76
76 71 16 75
16 76 57 75
38 76 56 16
Solution:
NIACL AO Mains Exam Reasoning Quiz: 23rd February 2019 |_8.1
Q9. What is the code for ‘Pradesh’?
38
76
75
83
Cannot be determined
Solution:
NIACL AO Mains Exam Reasoning Quiz: 23rd February 2019 |_8.1
Q10. For how many words, codes can be exactly determined?
five
six
seven
eight
None of these
Solution:
NIACL AO Mains Exam Reasoning Quiz: 23rd February 2019 |_8.1
Directions (11-15): Each of the questions below, consist of three statements numbered I, II and III. You have to decide whether the data provided in the statements are sufficient to answer the question. Read the three statements and Give answer
Q11. On which day in November is definitely Rohit’s mother’s birthday?
I. Rohit correctly remembers that her mother’s birthday is before eighteenth November but after twelfth November.
II. Rohit’s sister correctly remembers that her mother’s birthday is after fifteenth November but before nineteenth November.
III. Rohit’s Father correctly remembers that his wife’s birthday is after fourteenth November but before Seventeenth November.
If the data in statement I and II together are sufficient to answer the question, while the data in statement III are not required to answer the question.
If the data in statement I and III together are sufficient to answer the question, while the data in statement II are not required to answer the question.
If the data in statement II and III are sufficient to answer the question, while the data in statement I are not required to answer the question.
If the data in all three statements I, II and III together are necessary to answer the question.
If the data in all the statements, I, II and III even together are not sufficient to answer the question.
Solution:
From statement II,it is clear that Rohit’s mother birthday is on one of the day 16th 17th or on 18th of November.
From statement III it is clear that according to Rohit’s father’s wife’s (i.e. Rohit’s mother) birthday is on 15th or on 16th November.
Hence Rohit’ Mother’s birthday is on 16th November
Q12. Seven persons A, B, C, D, N, M and O sat around a round table then, what is the position of A with respect to C (Counted from left of C) in a round table seating arrangement if all of them are facing towards the center of the table?
I. A sits third to the left of M who is not the immediate neighbour of O. D sits immediate left of B. ?
II. N sits 2nd to the left of D. O sits on the immediate right of C. ?
III. M sits 2nd to the left of B, who is an immediate neighbour of A. C sits on the immediate right of O.
If the data in statement I and II together are sufficient to answer the question, while the data in statement III are not required to answer the question.
If the data in statement I and III together are sufficient to answer the question, while the data in statement II are not required to answer the question.
If the data in statement II and III are sufficient to answer the question, while the data in statement I are not required to answer the question.
If the data in all three statements I, II and III together are necessary to answer the question.
If the data in all the statements, I, II and III even together are not sufficient to answer the question.
Solution:
From statement I and II it is clear that A sits 4th to the left of C.
NIACL AO Mains Exam Reasoning Quiz: 23rd February 2019 |_13.1
Q13. There are seven persons i.e. A, B, C, M, K, T and L in a family. Find who is the mother of K?
I. M is the brother K. T is married to B. K is daughter of L. A is married to L.
II. M is the sibling of K. T is the father of A.
III. K is the only sister of M. A is the only child of B and his wife is L.
If the data in statement I and II together are sufficient to answer the question, while the data in statement III are not required to answer the question.
If the data in statement I and III together are sufficient to answer the question, while the data in statement II are not required to answer the question.
If the data in statement II and III are sufficient to answer the question, while the data in statement I are not required to answer the question.
If the data in all three statements I, II and III together are necessary to answer the question.
If the data in all the statements, I, II and III even together are not sufficient to answer the question.
Solution:
From Statement I and III we get the final solution, that L is the mother of K.
NIACL AO Mains Exam Reasoning Quiz: 23rd February 2019 |_14.1
Q14. In which direction and at what distance is point A is with respect to point M?
I. Point B is 2 m north of point N. point A is 2 m north of point B. Point E is 6 m east of point B.
II. Point I is 3m south of M. Point E is 3m west of point of I. Point B is 5m south of point E. Point A is to the west of point B.
III. Point E is 2m north of Point I. Point N is 6m west of point I. Point M is 2m south of point N.
If the data in statement I and II together are sufficient to answer the question, while the data in statement III are not required to answer the question.
If the data in statement I and III together are sufficient to answer the question, while the data in statement II are not required to answer the question.
If the data in statement II and III are sufficient to answer the question, while the data in statement I are not required to answer the question.
If the data in all three statements I, II and III together are necessary to answer the question.
If the data in all the statements, I, II and III even together are not sufficient to answer the question.
Solution:
Statement I and III together are necessary to answer the question.
NIACL AO Mains Exam Reasoning Quiz: 23rd February 2019 |_15.1
Q15.Six persons A, B, C, D, E and F are sitting in a row and all are facing north direction. What is the position of D from the left end?
I. A is sitting third from the right end. B and E are immediate neighbour of A.
II. B is sitting 2nd to the right of E but he is not the immediate neighbor of A. E is sitting one of the end. D is not the immediate neighbour of B.
III. E is sitting on the immediate right of A, who is second to the right of C. C and F are immediate neighbours.
If the data in statement I and II together are sufficient to answer the question, while the data in statement III are not required to answer the question.
If the data in statement I and III together are sufficient to answer the question, while the data in statement II are not required to answer the question.
If the data in statement II and III are sufficient to answer the question, while the data in statement I are not required to answer the question.
If the data in all three statements I, II and III together are necessary to answer the question.
If the data in all the statements, I, II and III even together are not sufficient to answer the question.
Solution:
Both statements I and III are necessary to answer the question.
NIACL AO Mains Exam Reasoning Quiz: 23rd February 2019 |_16.1
               




हिंदी में पढ़ें 

You may also like to Read:
NIACL AO Mains Exam Reasoning Quiz: 23rd February 2019 |_17.1         NIACL AO Mains Exam Reasoning Quiz: 23rd February 2019 |_18.1


Print Friendly and PDF

Leave a comment

Your email address will not be published. Required fields are marked *